PT31.S3.Q12 - several recent studies

Giant PandaGiant Panda Alum Member
edited October 2016 in Logical Reasoning 274 karma
Hi Gents and ladies,

Just a curious note on a confusion that I have. Question is here: https://7sage.com/lesson/serious-medical-condition-weaken-question/

I knew C is the one to support, but isn't A something wrong too?

In the lecture, JY says don't question the premise, but isn't A doing just that? There are study that confirms it and there is this 1 study that is recent which doesn't. Isn't this an attack?

Some clarifications will do wonders.

Thanks,

Panda

Comments

  • PositivePositive Alum Member
    426 karma
    Hi Panda,

    Indeed A does not go after the premise. For example, A doesn't say that the survey mentioned in the premise says the opposite.

    By adding a recent survey that says the opposite, A weakens the argument.

    Hope this helps.
  • ToxoplasmosisToxoplasmosis Alum Member
    233 karma
    I remember struggling with this question too. I think it's most helpful to think of it like this:

    Conclusion: Most people would want to be informed if they had a serious medical condition

    Why?

    Premise: Because a bunch of recent studies say this

    AC A. introduces more evidence that would cause us to question the conclusion we drew from the premises above. With the additional evidence in AC A. the argument becomes "most people want to be informed if they had a serious medical condition because some studies confirm this and some studies say the opposite of this" clearly a weaker argument.

    Hope this helps!
Sign In or Register to comment.